4a=5b+5 2a=3b+2. solve using the method of elimination
please help !*​

Answers

Answer 1

Answer:

a = 2.5, b = 1

Step-by-step explanation:

Given the 2 equations

4a = 5b + 5 → (1)

2a = 3b + 2 → (2)

Multiplying (2) by - 2 and adding the result to (1) will eliminate a

- 4a = - 6b - 4 → (3)

Add (1) and (3) term by term to eliminate a

0 = - b + 1 ( add b to both sides )

b = 1

Substitute b = 1 into either of the 2 equations and solve for a

Substituting into (2)

2a = 3(1) + 2 = 3 + 2 = 5 ( divide both sides by 2 )

a = 2.5

Answer 2

Answer:

a = 2.5

b = 1

Step-by-step explanation:

4a = 5b + 5 and 2a = 3b + 2

→ Multiply the second equation by 2

4a = 6b + 4

→ Subtract this new equation from the first one

0 = b - 1

→ Find b by adding 1 on both sides

b = 1

→ Substitute b into one of the equation

4a = ( 5 × 1 ) + 5

→ Simplify

4a = 10

→ Divide both sides by 4

a = 2.5


Related Questions

Does anyone know the answer

Answers

Answer:

It's False Because Hypotenuse will be the longest side of leg here

How much is the bill for a person who uses 560 kwh in a month?

Answers

Answer:

An electric company calculates a person's monthly bill from the number of kilowatt-hours (kWh), x, used, x 200 The function b(ar) = { 0.102, 0.15(x - 200) +20, 200 determines the bill How much is the bill for a person who uses 800 kWh in a month?

HElp Please its Pre-Calculus Matrix

Answers

The determinant is C. -541.

Using a cofactor expansion along the first row:

[tex]\begin{vmatrix}-8&-4&-1\\1&7&-3\\8&9&9\end{vmatrix} = -8\begin{vmatrix}7&-3\\9&9\end{vmatrix} + 4\begin{vmatrix}1&-3\\8&9\end{vmatrix} - \begin{vmatrix}1&7\\8&9\end{vmatrix} \\\\= -8(7\times9-(-3)\times9) + 4(1\times9-(-3)\times8) - (1\times9-7\times8) \\\\ = -8(63+27) + 4(9+24) - (9-56)\\\\= \boxed{-541}[/tex]

How much should 5x³+3x²-2x+1 be increased to get 6x² +7​

Answers

Answer:

-5x³ + 3x³ + 2x + 6

Step-by-step explanation:

if that is truly the complete problem description, then the only possible answer I can think of, that makes any sense :

what do I need to add to 5x³+3x²-2x+1 to get 6x²+7 ?

the answer is

6x²+7 - (5x³+3x²-2x+1) = 6x²+7-5x³-3x²+2x-1 =

= -5x³ + 3x³ + 2x + 6

Find the slope of the line containing the points (-3, -6) and (4, 7).

Answers

Answer:

[tex]\frac{13}{7}[/tex]

Step-by-step explanation:

→ State the slope formula

( y₂ - y₁ ) ÷ ( x₂ - x₁ )

→ Substitute in the values

( 7 - -6 ) ÷ ( 4 - -3 )

→ Simplify

[tex]\frac{13}{7}[/tex]

Answer:

slope = [tex]\frac{13}{7}[/tex]

Step-by-step explanation:

Calculate the slope m using the slope formula

m = [tex]\frac{y_{2}-y_{1} }{x_{2}-x_{1} }[/tex]

with (x₁, y₁ ) = (- 3, - 6) and (x₂, y₂ ) = (4, 7)

m = [tex]\frac{7-(-6)}{4-(-3)}[/tex] = [tex]\frac{7+6}{4+3}[/tex] = [tex]\frac{13}{7}[/tex]

Solve for X. Geometry

Answers

Answer:  " x = 10 " .

Step-by-step explanation:

Note the top "line segment" ; and the bottom "line"; have equal measurements:

The length of the top line segment is:

" (2x − 8) + 5 ."

The length of the bottom line is:  " (x + 7) " ;

__________

To solve for "x" ;

___________

(2x − 8) + 5 = (x + 7) ;

Rewrite as:

2x − 8 + 5 = x + 7 ;

Note: −8 + 5 = -3 ;

_______________

Rewrite the equation as:

 2x − 3 = x + 7 ;

We can subtract "x" from each side of the equation;

and add "3" to each side of the equation:

 2x − 3 − x + 3 = x + 7 − x + 3 ;

_______________

to get:

_______________

 x = 10 .

_______________

Let us check our answer

(2x − 8) + 5 =? (x + 7) ;

________

(2*10 − 8)  + 5 =? (10 + 7) ? ;

(20 − 8)  + 5 =? 17 ?

  12 + 5 =? 17 ?  Yes!

_______________

Best wishes in your academic pursuits!

_______________

Out of a class of 150 one third opted for Germany two fifth for Italian and rest for French. find how many opted for French​

Answers

Answer:

40

Step-by-step explanation:

Total is 150

Germany=1/3 *150 = 50

Italian= 2/5 * 150 = 60

Total - Germany- Italian= French

150 - 50 -60= 40

Identify the recursive formula for the sequence –3, 9, –27, 81, . . . .

Answers

Answer:

Hello,

Answer A

Step-by-step explanation:

[tex]a_n=(-3)*a_{n-1}\\a_1=-3\\Answer\ A[/tex]

Option A is correct -

[tex]f(n)=\left \{ {{f(1)=-3} \atop {f(n)=-3f(n-1)\;\;n > 1}} \right.[/tex]

We have a sequence : –3, 9, –27, 81, . . . .

We have to find the recursive formula for this sequence.

What is the formula to find the recursive of a Geometric sequence?

The formula to find the recursive of geometric sequence is -

[tex]a_{n} =ra_{n-1} \;\;\;\;for\;n\geq 2[/tex]

We have the following sequence -

–3, 9, –27, 81, . . . .

First let's see if it is a geometric sequence or not. For a sequence to be a geometric sequence -

[tex]\frac{9}{-3} =\frac{-27}{9} =\frac{81}{-27}= -3=r[/tex]

Hence, it is a geometric sequence with r = -3.

Substituting the value of r in the formula of recursive, we get -

[tex]a_{n} = -3a_{n-1}[/tex]    and      [tex]a(1)=-3[/tex]

Hence, Option A is correct.

To solve more questions on finding the recursive of a sequence, visit the link below -

brainly.com/question/24506976

#SPJ2

What are the simplest forms for red, blue, and green line

Answers

Answer:

Red line: y=-1/2x+1  Blue line: y=-1/3x-2  Green line: y=x

Step-by-step explanation:

f(x) red line m=-1/2 b=1

g(x) blue line m=-1/3 b=-2

h(x) green line m=1 b=0

The Bui family consists of 2 adults and 3 children. They spent $36 for zoo tickets. The equation 2x + 3y = 36 describes the cost for x adult tickets and y child tickets. Part of the equation is graphed below. What is the y-intercept?

On a coordinate plane, a line goes through points (6, 8) and (12, 4).
(0, 12)
(0, 18)
(12, 0)
(18, 0)

Answers

Answer: (0, 12)

Step-by-step explanation:

2x + 3y = 36

2(0) + 3y = 36

3y = 36

3/3 36\3

= 12

On edge

Answer:

a

Step-by-step explanation:

Select the best real-world situation that can be represented by 12 + = 13.50. Group of answer choices Mary wants to buy flowers and perfume. She has $13.50 to spend, but has already spent $12 on perfume. How much does she have left to spend on flowers? Rachel earns $12 per hour. How long does it take her to earn $13.50? Jane has $13.50 to spend. How many roses can she buy if they are $12 each? Sarah has $12 in her checking account, but has written a check for $13.50. How much is she overdrawn?

Answers

Answer:

Mary wants to buy flowers and perfume. She has $13.50 to spend, but has already spent $12 on perfume. How much does she have left to spend on flowers?

Step-by-step explanation:

12 + c = 13.50

OpruonnA seems to be the best, hence, Breaking down in real world terms :

13.50 = total cost

Number of variables added to make up total cost = 2 ; this represents the cost of the 2 items purchased ; flowers and perfume

Cost of perfume = 12

Cost of flowers = c

Total cost of items = $13.50

the product of 2x and 3 is 138. find the number x


Answers

Answer:

x = 23

General Formulas and Concepts:

Pre-Algebra

Equality Properties

Multiplication Property of Equality Division Property of Equality Addition Property of Equality Subtraction Property of Equality

Algebra I

Terms/Coefficients

Step-by-step explanation:

Step 1: Define

3(2x) = 138

Step 2: Solve for x

Simplify:                                                                                                             6x = 138[Division Property of Equality] Divide 6 on both sides:                                  x = 23

Out of a sample of 327 Americans, 245 said they had no interest in professional soccer.
(Data simulated from Carey & Kereslidze, 2007) A 95% confidence interval for the
proportion of Americans who have no interest in professional soccer is 0.70 to 0.80.
Suppose that another sample of 784 Americans was taken and asked the same question.
How would the width of the new confidence interval compare to the width of the
confidence interval based on the 327 women in the armed forces?
The width of the confidence interval based on the 784 women would be narrower than the
confidence interval based on the 327 women.
The width of the confidence interval based on the 784 women would be wider than the
confidence interval based on the 327 women.
The width of the confidence interval based on the 784 women would be the same as the
confidence interval based on the 327 women.
O No answer text provided.

Answers

Answer:

The width of the confidence interval based on the 784 women would be narrower than the  confidence interval based on the 327 women.

Step-by-step explanation:

In a sample with a number n of people surveyed with a probability of a success of [tex]\pi[/tex], and a confidence level of [tex]1-\alpha[/tex], we have the following confidence interval of proportions.

[tex]\pi \pm z\sqrt{\frac{\pi(1-\pi)}{n}}[/tex]

In which

z is the z-score that has a p-value of [tex]1 - \frac{\alpha}{2}[/tex].

The margin of error is:

[tex]M = z\sqrt{\frac{\pi(1-\pi)}{n}}[/tex]

From this, we have that the margin of error, and also the width, is inversely proportional to the sample size, that is, a larger sample size leads to a smaller margin of error and a narrower interval.

How would the width of the new confidence interval compare to the width of the  confidence interval based on the 327 women in the armed forces?

New interval: 784

Old interval: 327

Sample size increased, so the new interval will be narrower, and the correct answer is:

The width of the confidence interval based on the 784 women would be narrower than the  confidence interval based on the 327 women.

Find the value of the variable (x) to the nearest tenth.

Answers

Answer:

I think its ten

Step-by-step explanation:

an agent receives a commission of d% of the selling price of a house. if the commission amounted to 600, the selling price, in dollars was


Need help brainlist and will mark brainlist

Answers

Answer:

Selling price is $60000/d

Step-by-step explanation:

Let the selling price be say $x. Now the commission is d% of x, that is (d/100)*x=600. x=60000/d

Commission of d% amounts to 600 the selling price in dollers will be

Selling price = 600 * [tex]\frac{100}{d}[/tex] = 6000/d $

What are Selling price, Cost price, Profit &Loss ?

The price of an item which i have sold to someone is my selling price and for the person it is his/her cost price.

When my selling price is more than my cost price I make profit.

When my selling price is less than my cost price i make loss.

In the given question d% of commision amounts to 600$. Let us assume d% = 10 % for easy understanding.

[tex]\frac{percentage}{100}[/tex] * selling price = 600

[tex]\frac{d}{100}[/tex] * selling price = 600

Selling price = 600 * [tex]\frac{100}{d}[/tex] = 6000/d $

or, 60000/10 = 6000 $

Learn more about Profit and Loss here :

https://brainly.com/question/13934673

#SPJ2

ZABC and ZDEF are complementary angles.
What is the measure of x?
А,
С X = [?]°
549
D
B
Х
>F

Answers

Answer:

x = 36

Step-by-step explanation:

Complementary angles add to 90 degrees

54+x = 90

x = 90-54

x = 36

Answer:

IF ABC and DEF are complementary then ABC + DEF = 90°

Therefore, DEF = 90° -ABC

Since DEF = 54° and DEF = x then

x = 90° -54°

x = 36°

Step-by-step explanation:

graph the line that passes through the points (-3,-7) and (-2,-7) and determine the equation of the line

Answers

Answer:

y = -7

Step-by-step explanation:

y2 - y1 / x2 - x1

-7 - (-7) / -2 - (-3)

0/1

= 0

y = 0x + b

-7 = 0(-2) + b

-7 = 0 + b

-7 = b

If a graph is a line, it will always have a domain of all real numbers.
True or False

Answers

Answer:

i think its true.

Step-by-step explanation:

Hope it helps!

Fiona y sus amigas fueron a una tienda de batidos. Pidieron tres batidos de fresa y dos batidos de proteínas por un total de $ 24,00. La semana pasada pidieron un batido de fresa y cuatro batidos de proteínas por un total de $ 25,50. El sistema que representa sus compras se encuentra a continuación. x + 4 y = 25,50 3 x + 2 y = 24,00 ¿Cuál es el precio de un batido de proteínas? $ 3,75 $ 4.50 $ 4.95 $ 5.25

Answers

Answer:

Un batido de proteínas cuesta $ 5,25.

Step-by-step explanation:

El sistema de ecuaciones descrito al final del enunciado coincide con lo descrito al comienzo del mismo, de modo que podemos concentraron casi exclusivamente en la resolución del sistema en cuestión, la cual se describe a continuación:

Este es el sistema a resolver:

[tex]x + 4\cdot y = 25,50[/tex] (1)

[tex]3\cdot x + 2\cdot y = 24,00[/tex] (2)

Donde [tex]x[/tex] es el coste de un batido de fresa y [tex]y[/tex] corresponde al coste de un batido de proteínas:

1) Despejamos [tex]x[/tex] en (1):

[tex]x = 25,50 - 4\cdot y[/tex]

2) Eliminamos [tex]x[/tex] en (2):

[tex]3\cdot (25,50 - 4\cdot y) + 2\cdot y = 24,00[/tex]

[tex]76,50 - 12\cdot y +2\cdot y = 24,00[/tex]

3) Simplificamos y despejamos [tex]y[/tex]:

[tex]10\cdot y = 52,50[/tex]

[tex]y = 5,25[/tex]

Un batido de proteínas cuesta $ 5,25.

Find the Segment.I need help ASAP

Answers

Answer:

Does the answer help you?

Answer:

x = 65 units

Step-by-step explanation:

Using the Altitude- on- Hypotenuse theorem

(leg of large triangle)² = (part of hypotenuse below it) × (whole hypotenuse), so

x² = 25 × (25 + 144) = 25 × 169 = 4225 ( take square root of both sides )

x = [tex]\sqrt{4225}[/tex] = 65 units

Solve inequality problem (urgent help please)

Answers

Answer:

B

Step-by-step explanation:

|3m-2|=>1

1 <= 3m-2 or 3m-2 <= -1

3 <= 3m or 3m <= 1

1 <= m or m <= (1/3)

Determine the probability (as a decimal rounded to the nearest hundredth) of a normal random variable with a mean of three and a standard deviation of four realizing a value that is strictly greater than 1.9

Answers

Answer:

Step-by-step explanation:

On a normal distribution table with z scores of 0 as the mean and the standard deviations going to the right and to the left, 1.9 on the normal curve where 3 is the mean falls at -.275 on the normal curve where 0 is the mean. I used the normal distribution z table for negative values to get that .39165 lies to the left of -.275, which means that 1 - .39165 of the data lies to the right.

The probability that the value is greater than 1.9 is .60835, or as a percentage, 60.835%.

PLZ HELP!! ASAP!! IT"S FROM KHAN!! NO FILES!!

Answers

Answer:

The slope is (1/4)

Step-by-step explanation:

Slope is calculated for the points (5,6) and (1,5). Slope is (6-5)/(5-1)=1/4

Answer: m = 10/3

Step-by-step explanation:

(0.3, 1) (0.6, 2)

(2-1)/(0.6-0.3)=

1/0.3 = 10/3

Hotdogs come in packages of 10, while rolls come in either 8 or 12. What is the smallest number of packages of each you have to buy in order to have the same number of hotdogs and rolls?
Please explain the answer.

Answers

if the rolls come in 8

then we check which multiples 8 and 10 have in common which is 40

8,16,24,32,40

10,20,30,40

which is 5 packages of rolls and 4 pack of hot dogs

if the rolls are 12

we check the multiples which they have in common

12,24,36,48,60

10,20,30,40,50,60

which is 5 packages of rolls and 6 pack of hot dogs

Pls mark as brainliest


Two supplementary angles measure (7x + 11) and (14x + 1)º. What is
the measure of the smaller angle?
(1) 40.5°
(2) 67°
(3) 108°
(4) 123°

Answers

Answer:

67 degrees (Answer 2)

Step-by-step explanation:

These two angles must sum up to 180 degreess because they are supplementary.  Then:

7x + 11 + 14x + 1 = 180, or

21x + 12 = 180, or

21x = 168

Solving for x, we divide both sides by 21.  This results in x = 8.

The measure of the smaller angle is 7x + 11, which here is 7(8) + 11 = 67 degrees (Answer 2).

asnwer question in screenshot pls

Answers

Answers:first box = 0 second box = 0 third box = 4fourth box = -3

It's not a typo that 0 is written twice.

==============================================================

Explanation:

This is considered a step function because it resembles steps or a staircase. The vertical pieces aren't drawn to allow it to remain a function (or else it fails the vertical line test). Also there should be open holes at each left endpoint.

Recall that any horizontal line has a slope of 0 due to the fact that the rise is 0

slope = rise/run

The rise takes care of the vertical change in position. But if the line doesn't go up or down, then rise = 0 leads to slope = 0.

In short, the slope of each horizontal portion is 0. It doesn't matter which sub-section you focus on.

This is why you'll type 0 into the first two boxes.

------------------

As for the remaining two boxes, you'll be looking where the graph is the highest and the lowest to determine the max and min y values respectively.

We see that y peaks at y = 4 at the very top. So you'll type 4 into the third box.

Furthermore, we see that the lowest part of the graph is when y = -3. Therefore, this is the minimum aka smallest y value and you'll type -3 into the fourth box.

Is this a function or no ?

Answers

It is not a function look at the picture it will show you

H = 5 ft
Which is the formula
for the volume
of a cylinder?
A. V = a^2 + b^2 + c^2
B. V = (area of the base)(radius)
C. V = (area of the base)(height)

Answers

The answer is c!!! I think???

Answer:  C.  V = (area of the base)(height)

This applies to any prism. A cylinder is effectively a prism but the base is a circle rather than a polygon.

The cylindrical volume formula you may be more familiar with is

V = pi*r^2*h

note how the pi*r^2 portion is the area of the circular base. So we can replace the pi*r^2 with "area of the base" to end up with choice C.

could you pls teach me this question ​

Answers

Answer:

if it's area the area would be the length of the diameter of the semi circle which is the height of the triangle multiplied by the base of the triangle to find the area of the triangle then to find the area of the circle would be pie multiplied by the diameter divided by two squared

[tex]\pi \ \times 7 {}^{2} [/tex]

then the total area would be the area of the triangle plus the area of the half circle

For the system of equations:
-4s + 2t = -5
8s - 6t=7
What is the value of s?

Answers

Answer:

s = 2

Step-by-step explanation:

I'll be using the substitution method.

Other Questions
if Create an exponential function, and then create a second exponential function that shifts your original function down 7 units A 40-year-old male vegetarian is diagnosed with folate deficiency anemia. He reports that he is an alcoholic. Which of the following factors put him at greatest risk for developing his disease? a. Being vegetarian b. Being alcoholic c. Age d. Gender what the movement of the earth around the sun ANALYZE How could scientists use data from tree rings to learn more about ecosystemsin the past? Use evidence from the tree ring in Figure 2a to support your answer. Required-: JOURNAL ENTRYPaid Rs. 28,000 to Sansee stores in fullsettlement of Rs. 30,000 by cheque please help Me with this.I'm still confused in it. En una radio, 1/8 del dia esta destinado a emitir publicidad. Del total de publucidad, 3/5 corresponden a comerciales de univercidades. En esta radio,que fraccion del tiempo diario corrsponde a publicidad de univercidades. Can anyone decipher these? There a bit random. Maybe binary but if there are other ways please include them. The most common carrier of communicable diseases are a.cockroach b.housefly c.mosquito d.spider What does equal protection under law mean Income Statement, Direct and Indirect Cost Concepts, Service CompanyLakeesha Barnett owns and operates a package mailing store in a college town. Her store, Send It Packing, helps customers wrap items and send them via UPS, FedEx, and the USPS. Send It Packing also rents mailboxes to customers by the month. In May, purchases of materials (stamps, cardboard boxes, tape, Styrofoam peanuts, bubble wrap, etc.) equaled $11,450; the beginning inventory of materials was $1,050, and the ending inventory of materials was $950. Payments for direct labor during the month totaled $25,570. Overhead incurred was $18,130 (including rent, utilities, and insurance, as well as payments of $14,050 to UPS and FedEx for the delivery services sold). Since Send It Packing is a franchise, Lakeesha owes a monthly franchise fee of 5 percent of sales. She spent $2,750 on advertising during the month. Other administrative costs (including accounting and legal services and a trip to Dallas for training) amounted to $3,650 for the month. Revenues for May were $102,100.Required:1. What was the cost of materials used for packaging and mailing services during May?2. What was the prime cost for May?3. What was the conversion cost for May?4. What was the total cost of services for May?5. Prepare an income statement for May.6. Of the overhead incurred, is any of it direct? Indirect? Explain. Navassa Island is a U.S. territory. It is southeast of Florida and west of Puerto Rico. Which letter on the map represents the location of Navassa Island?A.B.C.D. A committee raised 7/9 of their target goal last year and another 1/9 of the goal this year. What fraction of their goal has been raised with nonfiction, looking at the title of the work and the table of contents is helpful 3. A bacterial isolate from a urine specimen was grown in culture, Gram stained, and then tested for its ability to ferment sugars and hydrolyze various subtrates. What approach to bacterial identification is this an example of Consulta om de yue el siguiente come HORIZONTALES VERTICALES 3.- ca de Copenic porque bregata, e uno de Emmani denganayamalisaregates 6.2 dractisenaremma, e capes az embere ganes movimento de los planetas y cred la teca 2- En camere decorado per les 7 - Los esos desconden het diese 4- Proposta gestie E-Seene que esde centro de Sisteme 5-Ten el Tenes Solar, que la Tea y les dents are firma cabo de ese ya su alrededor de wreeds, que esta gua sobre si misma una salyfes tenis areta we al dia quien me ayuda es para hoy Christopher Columbus born in Genoa never did reach mainland North America. How did the lack of executive and judicial branches weaken the national government under the articles of confederation? What value of g makes the equation true? A rescue plane spots a survivor 132 m directly below and releases an emergency kit with a parachute. If the package descends at a constant vertical acceleration of 6.89 m/s2 the initial plane horizontal speed was 86.9 m/s, how far away from the survivor will it hit the waves